Area under curve problem again

Click For Summary
The discussion revolves around finding the equation of a curve given its gradient, which is 3x² - 6x + 4, and that it passes through the point (-1, 0). The integrated equation of the curve is x³ - 3x² + 4x + 8. The main challenge is calculating the area enclosed by the curve, the x-axis, and the vertical lines at x=1 and x=2. The user initially integrated the gradient correctly but is confused about the area calculation, mistakenly believing their result of 2 was incorrect. Clarification is provided that integrating the gradient from x=1 to x=2 yields the correct area of 2.
lionely
Messages
574
Reaction score
2

Homework Statement


Find the equation of the curve which passes through the point (-1,0) and whose gradient at
any point (x,y) is 3x2-6x+4. Find the area enclosed by the curve, the axis of x and the ordinates x=1 and x=2.


. The attempt at a solution

I integrated and got the equation of the curve to be x3 -3x2 +4x +8

Now to find the area I'm not sure what to do, I tried to sketch a graph but I couldn't.

I tried integrating from x=1 to x=2 and I got 2, which isn't the answer in the book :S.

Help greatly appreciated.
 
Physics news on Phys.org
lionely said:

Homework Statement


Find the equation of the curve which passes through the point (-1,0) and whose gradient at
any point (x,y) is 3x2-6x+4. Find the area enclosed by the curve, the axis of x and the ordinates x=1 and x=2.


. The attempt at a solution

I integrated and got the equation of the curve to be x3 -3x2 +4x +8

Now to find the area I'm not sure what to do, I tried to sketch a graph but I couldn't.

I tried integrating from x=1 to x=2 and I got 2, which isn't the answer in the book :S.

Help greatly appreciated.

Which equation did you integrate? You are given the gradient of the curve at any point (x,y). You found the curve. Find the area closed by this curve and the x-axis.
 
x^3 - 3x^2 +4x +8
 
lionely said:
x^3 - 3x^2 +4x +8
Show the steps you made when you integrated this from x=1 to x=2 to result in the value 2.
 
Last edited:
lionely said:
x^3 - 3x^2 +4x +8

Integrate it again from x=1 to x=2. If you integrate 3x^2-6x+4 from x=1 to x=2, then you get the value 2. Knowing this, I asked you which equation did you integrate.
 
Sorry I intregrated 3x^2-6x+4 for x=1 to x=2 and got 2
 
Question: A clock's minute hand has length 4 and its hour hand has length 3. What is the distance between the tips at the moment when it is increasing most rapidly?(Putnam Exam Question) Answer: Making assumption that both the hands moves at constant angular velocities, the answer is ## \sqrt{7} .## But don't you think this assumption is somewhat doubtful and wrong?

Similar threads

Replies
5
Views
2K
  • · Replies 7 ·
Replies
7
Views
2K
  • · Replies 2 ·
Replies
2
Views
2K
  • · Replies 13 ·
Replies
13
Views
5K
Replies
2
Views
2K
  • · Replies 8 ·
Replies
8
Views
2K
  • · Replies 2 ·
Replies
2
Views
2K
  • · Replies 12 ·
Replies
12
Views
2K
  • · Replies 6 ·
Replies
6
Views
2K
  • · Replies 2 ·
Replies
2
Views
2K